124. Limit by L'Hopital's Rule: with logarithm, form 0 times infinity

Поділитися
Вставка
  • Опубліковано 18 гру 2017
  • ▼ IMPORTANT ▼ Solved example of limit using L'Hopital's rule (L'Hôpital's rule, L'Hospital's rule), in the case in which x tends to zero and we have a product of x times natural logarithm, of the form zero times infinity, writing the product as a division (quotient), to have the form 0/0 or infinity over infinity and apply the rule deriving the logarithmic and rational function and calculating the limit step by step.
    #limit #lhopital #calculation
    ----------
    ** IMPORTANT LINKS **
    Limits Course: • Límites
    Special Videos: • Members-only videos
    Mathematics Review Course (Pre-University) • Curso de Matemáticas
    ----------
    ** SEE ALL MY COURSES HERE **
    / arquimedes1075
    ----------
    ** BIBLIOGRAPHY **
    - Calculus of a variable, James Stewart
    - Calculus, M. Spivak
    - 5000 mathematical analysis problems, B. Demidovich
    - Calculus, Granville
    - Simplified Mathematics, by Conamat
    ----------
    ** DONATIONS **
    - Paypal: www.paypal.com/cgi-bin/webscr...
    - Channel memberships: / @matefacilyt
    - Patreon: / matefacil
    ----------
    ** MY OTHER CHANNELS AND SOCIAL NETWORKS **
    - Physics Channel: / matefacil - física
    - Videogame Channel: / @matefacily
    - Twitch: / matefacil
    - MateFacil App: educup.io/apps/matefacil
    - Facebook (Page): / arquimedes1075
    - Twitter: @Matefacilx
    - Instagram: @Matefacilx
    - Discord: / discord
    ----------
    #Matefacil #Matematicas #Math #tutorial #tutor #tutoriales #profesor
    ----

КОМЕНТАРІ • 49

  • @MateFacilYT
    @MateFacilYT  2 роки тому +2

    *¡Hola! ¿Necesitas ayuda con tus ejercicios?* Escríbeme en cualquiera de mis redes sociales:
    Facebook.com/MatefacilYT
    Telegram: t.me/matefacilgrupo
    twitter.com/matefacilx
    instagram.com/matefacilx
    tiktok.com/@matefacilx

  • @p2p2p2p2p2p
    @p2p2p2p2p2p 3 роки тому +14

    ¡Mi español es tan malo, pero este video me ayudó a resolver mi problema! ¡¡¡Muchas gracias!!!

  • @crsxlndl3555
    @crsxlndl3555 6 років тому +2

    Gracias ... muy útil !

  • @fisicauniversitaria
    @fisicauniversitaria 2 роки тому +1

    muy útil , como todos tus vídeos.Gracias.

    • @MateFacilYT
      @MateFacilYT  2 роки тому +1

      Me da mucho gusto haberte ayudado!

  • @Munaam1989
    @Munaam1989 6 років тому +2

    Gracias

  • @juandeandrade5269
    @juandeandrade5269 2 роки тому +1

    gracias

  • @YagmurCanalp
    @YagmurCanalp 4 роки тому +5

    thank youuu , I 'am from turkey. your video very good

    • @makeymator4105
      @makeymator4105 4 роки тому

      porque no miras a un turco haciendo el video amigo te la re complicas

    • @cesargomez4440
      @cesargomez4440 4 роки тому +2

      @@makeymator4105 quizás porque no hay youtubers turcos enseñando matemáticas. Si incluso hay brasileños en estos vídeos.y a decir verdad.uno con una base de matemáticas entiende estos vídeos sin audio o en el caso del turco en idioma español.

  • @fabrizioale3733
    @fabrizioale3733 3 роки тому +4

    Una pregunta, si quisiéramos calcular el limite cuando x tiende a 0 pero sin especificar si se trata por derecha o izquierda ¿El valor seguiría siendo el mismo? Entiendo que el límite lateral por izquierda no puede calcularse puesto que no está definido en el dominio de la función pero no se a qué conclusión llegar con respecto a la existencia o no del límite.

    • @ni10co9
      @ni10co9 3 місяці тому

      buena pregunta. Creo que si buscamos el limite por si mismo, no existe, pero si se puede trabajar si estamos integrando (creo)

  • @gian7470
    @gian7470 6 років тому +8

    Muy buena forma de explicar, realmente se agradecen muchisimo estos videos. Es usted de méxico cierto? Estudio alguna ingeniería?

    • @MateFacilYT
      @MateFacilYT  6 років тому +15

      ¡Hola!
      Sí, soy de México. Estudié licenciatura en ciencias Físico Matemáticas.
      Saludos.

  • @samuelquintero6867
    @samuelquintero6867 2 роки тому

    Hola muy buena tu explicación 💯, pero tengo una duda cuál sería el resultado si tengo Logaritmo natural de infinito positivo entre infinito negativo?

  • @victorlapuente6292
    @victorlapuente6292 4 роки тому +1

    yo lo hice porque me sabia la función del logaritmo y viendo la función lo saque. Pero gracias por la explicación

  • @fabiohoyosjojoa8424
    @fabiohoyosjojoa8424 3 роки тому

    Hola que hago si tengo en límite cuando x tiende a cero uno menos coseno de x por lnx

  • @andreanoeliamenegazzisanz421

    En mi carrera no me permiten usar regla de L'hopital. Cómo podemos hacer ese mismo límite usando Orden de Magnitud?

  • @asdf-jc3tb
    @asdf-jc3tb 4 роки тому

    si le quitas el x que multiplique al logaritmo, como saldría?

  • @cristianmendoza7212
    @cristianmendoza7212 4 роки тому +1

    ¿Se puede decir que el límite existe o el límite lateral nada más?

  • @DvdAglr
    @DvdAglr 3 роки тому

    Y si solo tengo ln x cuandl x tiende a 0+ ¿cómo calculo el límite? Ayuda u.u

  • @gonzaloqueondab9053
    @gonzaloqueondab9053 5 років тому

    Una pregunta ¿ por que derivaste el denominador y el numerador por separado ? ¿ que no las fracciones se derivan meriante una formula aparte ?

    • @MateFacilYT
      @MateFacilYT  5 років тому +1

      Esa es la regla de L'Hopital. Te recomiendo ver los videos anteriores para que entiendas mucho mejor el tema. El enlace a la lista completa esta en la descripción del video.

    • @gonzaloqueondab9053
      @gonzaloqueondab9053 5 років тому

      @@MateFacilYT muchas gracias

  • @christiandiaz2821
    @christiandiaz2821 11 місяців тому +1

    Buenos días profe, ¿Por qué podemos utilizar y cambiar ese producto de funciones? ¿Sería posible enviar la x al denominador como x^-1?. Muchas gracias🎉

    • @MateFacilYT
      @MateFacilYT  11 місяців тому +1

      Hola! Es simplemente álgebra.
      ab = a/(1/b) = b/(1/a)

  • @ramiromagallanes5094
    @ramiromagallanes5094 Рік тому

    Hola que tal, y si en vez de tener x, tengo raíz de x??

  • @juansantos-ts9nt
    @juansantos-ts9nt 2 роки тому

    como se haria en el caso de tener x al cuadrado Ln (x)

  • @fabiancabrera1004
    @fabiancabrera1004 4 роки тому

    hola, no entiendo de donde sale las dos formas que dices que podemos usar. es una propiedad de los límites ?

    • @brauogs
      @brauogs 4 роки тому

      No, son reglas de derivación: la derivada de logaritmo y la derivada usando la regla de la potencia. 2 semanas tarde, pero sí te sirve lol.

  • @sabasmoreno6705
    @sabasmoreno6705 Рік тому

    Que sucede si sumamos cero infinitas veces el resultado es cero o es una indeterminación?

    • @MateFacilYT
      @MateFacilYT  Рік тому +2

      Es cero, ya que una suma infinita es el límite de las sumas parciales, y en ese caso todas las sumas parciales son cero, así que se trata de la sucesión constante cero, la cual tiene como límite cero.

  • @cristiandioses7567
    @cristiandioses7567 6 років тому +2

    Hola que tal? existe otro metodo para resolver el limite sin aplicar la regla de L‘ Hopital?

    • @mateoloto4313
      @mateoloto4313 Рік тому

      4 años tarde pero si por sustitucion

    • @Nytram96
      @Nytram96 Рік тому

      @@mateoloto4313 Por sustitución cómo sería? Reemplazando x=1/z? con z tendiendo a infinito? No le encuentro la vuelta hacerlo de esa forma

    • @mateoloto4313
      @mateoloto4313 Рік тому

      @@Nytram96 fue hace un mes que hize ese problema ahora lo veo de vuelta a ver cómo es que me quedo

    • @mateoloto4313
      @mateoloto4313 Рік тому

      De todas formas es más complicado y más largo que LHopital,te recomiendo que te aprendas bien está herramienta que sirve un monton

  • @cristopfercordero5763
    @cristopfercordero5763 9 місяців тому

    Gracias, porque en el libro de purcell, no vi que nada de lo que aplicaste saliera.

  • @danielguerrero2147
    @danielguerrero2147 5 років тому

    y si es cero por la izquierda?

    • @MateFacilYT
      @MateFacilYT  5 років тому +8

      En este caso no tiene sentido, ya que el logaritmo solo existe para numeros positivos, por eso es limite por derecha.

  • @DoguiGM
    @DoguiGM Місяць тому +1

    julio 2024... crack

  • @thenicicles708
    @thenicicles708 5 років тому

    ¿Por qué 0 x +/-infinito es una indeterminación?

    • @MateFacilYT
      @MateFacilYT  5 років тому +3

      Porque dependiendo de las funciones que originan esa expresion, se puede obtener un valor u obtener otro valor. Es decir, no se obtiene como resultado un único valor.
      Y para obtener el valor correcto, hay que analizar el limite mediante regla de L'Hopital como explico en el video.
      Saludos.

    • @thenicicles708
      @thenicicles708 5 років тому +1

      @@MateFacilYT Gracias :)

  • @nicolasgu_
    @nicolasgu_ 5 років тому

    x-lnx , x tendiendo a infinito como lo hago ?

  • @rolandopena4220
    @rolandopena4220 5 років тому +2

    gracias por los videos si ayudan, pero la manera de resolverlo que ustedes tienen con formulas no es eficiente en la vida real, porque no creo que nosotros los estudiantes nos vayamos a aprender tantas formulas y tan parecidas, cuando hay que aprender a integrar, a derivar, a usar L´Hospital, a averiguar el caracter de las series y tantas cosas mas lo mas eficiente es tener un metodo que funcione con la mayoria, como integrar por partes, sustitucion trigonometrica, fracciones parciales ya que estos metodos se pueden aplicar en muchos casos, no es una formula que solo me servira para un problema en especifico. espero haberme hecho entender. gracias por el video

  • @FABIANALEJANDROTORVISCOCHUQUIM
    @FABIANALEJANDROTORVISCOCHUQUIM 2 місяці тому

    Sin l'hopital

  • @jezabelhsr
    @jezabelhsr 3 роки тому

    Y si ahora es x(ln x)^n...?!?